Finding the closed form of a generating function












2












$begingroup$


Given that $k$ is a positive integer and $f(x)$ is the generating function of the sequence $(b_0,b_1,b_2,...)$ where $b_n = {n choose k};, forall ;n$, show that: $$f(x)=frac{x^k}{(1-x)^{k+1}}$$
I tried writing a few terms of $f(x)$:$$f(x)=x^k+{k+1 choose k}x^{k+1}+{k+2 choose k}x^{k+2}+...$$$$;;;;;;;;;=x^kleft(1+{k+1 choose k}x^1+{k+2 choose k}x^2+...right)$$$$=x^kcdot h(x);;;;;;;;;;;;;;;;;;;;;;;;;;;;;;;;;;;; $$
Where $h(x)$ is defined as the expression in parenthesis, then I tried some manipulation, for example I computed $h(x)-xh(x)$, since: ${k+n+1 choose k}-{k+n choose k}={k+n choose k-1}$ we have:



$h(x)-xh(x)=1+{k choose k-1}x+{k+1 choose k-1}x^2+...$



But I'm stuck, not sure how I should procede










share|cite|improve this question









$endgroup$

















    2












    $begingroup$


    Given that $k$ is a positive integer and $f(x)$ is the generating function of the sequence $(b_0,b_1,b_2,...)$ where $b_n = {n choose k};, forall ;n$, show that: $$f(x)=frac{x^k}{(1-x)^{k+1}}$$
    I tried writing a few terms of $f(x)$:$$f(x)=x^k+{k+1 choose k}x^{k+1}+{k+2 choose k}x^{k+2}+...$$$$;;;;;;;;;=x^kleft(1+{k+1 choose k}x^1+{k+2 choose k}x^2+...right)$$$$=x^kcdot h(x);;;;;;;;;;;;;;;;;;;;;;;;;;;;;;;;;;;; $$
    Where $h(x)$ is defined as the expression in parenthesis, then I tried some manipulation, for example I computed $h(x)-xh(x)$, since: ${k+n+1 choose k}-{k+n choose k}={k+n choose k-1}$ we have:



    $h(x)-xh(x)=1+{k choose k-1}x+{k+1 choose k-1}x^2+...$



    But I'm stuck, not sure how I should procede










    share|cite|improve this question









    $endgroup$















      2












      2








      2


      0



      $begingroup$


      Given that $k$ is a positive integer and $f(x)$ is the generating function of the sequence $(b_0,b_1,b_2,...)$ where $b_n = {n choose k};, forall ;n$, show that: $$f(x)=frac{x^k}{(1-x)^{k+1}}$$
      I tried writing a few terms of $f(x)$:$$f(x)=x^k+{k+1 choose k}x^{k+1}+{k+2 choose k}x^{k+2}+...$$$$;;;;;;;;;=x^kleft(1+{k+1 choose k}x^1+{k+2 choose k}x^2+...right)$$$$=x^kcdot h(x);;;;;;;;;;;;;;;;;;;;;;;;;;;;;;;;;;;; $$
      Where $h(x)$ is defined as the expression in parenthesis, then I tried some manipulation, for example I computed $h(x)-xh(x)$, since: ${k+n+1 choose k}-{k+n choose k}={k+n choose k-1}$ we have:



      $h(x)-xh(x)=1+{k choose k-1}x+{k+1 choose k-1}x^2+...$



      But I'm stuck, not sure how I should procede










      share|cite|improve this question









      $endgroup$




      Given that $k$ is a positive integer and $f(x)$ is the generating function of the sequence $(b_0,b_1,b_2,...)$ where $b_n = {n choose k};, forall ;n$, show that: $$f(x)=frac{x^k}{(1-x)^{k+1}}$$
      I tried writing a few terms of $f(x)$:$$f(x)=x^k+{k+1 choose k}x^{k+1}+{k+2 choose k}x^{k+2}+...$$$$;;;;;;;;;=x^kleft(1+{k+1 choose k}x^1+{k+2 choose k}x^2+...right)$$$$=x^kcdot h(x);;;;;;;;;;;;;;;;;;;;;;;;;;;;;;;;;;;; $$
      Where $h(x)$ is defined as the expression in parenthesis, then I tried some manipulation, for example I computed $h(x)-xh(x)$, since: ${k+n+1 choose k}-{k+n choose k}={k+n choose k-1}$ we have:



      $h(x)-xh(x)=1+{k choose k-1}x+{k+1 choose k-1}x^2+...$



      But I'm stuck, not sure how I should procede







      generating-functions






      share|cite|improve this question













      share|cite|improve this question











      share|cite|improve this question




      share|cite|improve this question










      asked Jan 25 at 15:54









      Spasoje DurovicSpasoje Durovic

      38210




      38210






















          2 Answers
          2






          active

          oldest

          votes


















          2












          $begingroup$

          Here is a variation based upon the coefficient of operator $[x^n]$ which denotes the coefficient of $x^n$ of a series.




          We obtain
          begin{align*}
          color{blue}{b_n=[x^n]f(x)}&=[x^n]frac{x^k}{(1-x)^{k+1}}\
          &=[x^{n-k}](1-x)^{-k-1}tag{1}\
          &=[x^{n-k}]sum_{j=0}^infty binom{-k-1}{j}(-x)^jtag{2}\
          &=[x^{n-k}]sum_{j=0}^infty binom{k+j}{j}x^jtag{3}\
          &=binom{n}{n-k}tag{4}\
          &,,color{blue}{=binom{n}{k}}tag{5}
          end{align*}



          and the claim follows.




          Comment:




          • In (1) we apply the rule $[x^{p-q}]A(x)=[x^p]x^qA(x)$.


          • In (2) we apply the binomial series expansion.


          • In (3) we use the binomial identity $binom{-p}{q}=binom{p+q-1}{q}(-1)^q$.


          • In (4) we select the coefficient of $x^{n-k}$.


          • In (5) we use the binomial identity $binom{p}{q}=binom{p}{p-q}$.







          share|cite|improve this answer









          $endgroup$





















            1












            $begingroup$

            HINT:



            Inductive hypothesis
            begin{eqnarray*}
            f_k(x)= sum_{n=k}^{infty} binom{n}{k} x^n = frac{x^k}{(1-x)^{k+1}}
            end{eqnarray*}

            and use
            begin{eqnarray*}
            binom{n}{k} + binom{n}{k+1}= binom{n+1}{k+1}.
            end{eqnarray*}



            More detail available on request.



            Further Hint :



            begin{eqnarray*}
            f_{k+1}(x) &=& sum_{n=k+1}^{infty} binom{n}{k+1} x^n = sum_{n=k}^{infty} binom{n+1}{k+1} x^{n+1} \
            &=& sum_{n=k}^{infty} binom{n}{k} x^{n+1} + sum_{n=k}^{infty} binom{n}{k+1} x^{n+1} \ cdots
            end{eqnarray*}






            share|cite|improve this answer











            $endgroup$













            • $begingroup$
              From this I got $f_{k+1}(x)=frac{x^k}{(1-x)^{k+2}}$ so $f_{k+1}(x)=frac{1}{(1-x)}f_k(x)$ I never used induction on generating functions so I'm not sure what I'm supposed to do or if I did this correctly
              $endgroup$
              – Spasoje Durovic
              Jan 25 at 16:39













            Your Answer





            StackExchange.ifUsing("editor", function () {
            return StackExchange.using("mathjaxEditing", function () {
            StackExchange.MarkdownEditor.creationCallbacks.add(function (editor, postfix) {
            StackExchange.mathjaxEditing.prepareWmdForMathJax(editor, postfix, [["$", "$"], ["\\(","\\)"]]);
            });
            });
            }, "mathjax-editing");

            StackExchange.ready(function() {
            var channelOptions = {
            tags: "".split(" "),
            id: "69"
            };
            initTagRenderer("".split(" "), "".split(" "), channelOptions);

            StackExchange.using("externalEditor", function() {
            // Have to fire editor after snippets, if snippets enabled
            if (StackExchange.settings.snippets.snippetsEnabled) {
            StackExchange.using("snippets", function() {
            createEditor();
            });
            }
            else {
            createEditor();
            }
            });

            function createEditor() {
            StackExchange.prepareEditor({
            heartbeatType: 'answer',
            autoActivateHeartbeat: false,
            convertImagesToLinks: true,
            noModals: true,
            showLowRepImageUploadWarning: true,
            reputationToPostImages: 10,
            bindNavPrevention: true,
            postfix: "",
            imageUploader: {
            brandingHtml: "Powered by u003ca class="icon-imgur-white" href="https://imgur.com/"u003eu003c/au003e",
            contentPolicyHtml: "User contributions licensed under u003ca href="https://creativecommons.org/licenses/by-sa/3.0/"u003ecc by-sa 3.0 with attribution requiredu003c/au003e u003ca href="https://stackoverflow.com/legal/content-policy"u003e(content policy)u003c/au003e",
            allowUrls: true
            },
            noCode: true, onDemand: true,
            discardSelector: ".discard-answer"
            ,immediatelyShowMarkdownHelp:true
            });


            }
            });














            draft saved

            draft discarded


















            StackExchange.ready(
            function () {
            StackExchange.openid.initPostLogin('.new-post-login', 'https%3a%2f%2fmath.stackexchange.com%2fquestions%2f3087246%2ffinding-the-closed-form-of-a-generating-function%23new-answer', 'question_page');
            }
            );

            Post as a guest















            Required, but never shown

























            2 Answers
            2






            active

            oldest

            votes








            2 Answers
            2






            active

            oldest

            votes









            active

            oldest

            votes






            active

            oldest

            votes









            2












            $begingroup$

            Here is a variation based upon the coefficient of operator $[x^n]$ which denotes the coefficient of $x^n$ of a series.




            We obtain
            begin{align*}
            color{blue}{b_n=[x^n]f(x)}&=[x^n]frac{x^k}{(1-x)^{k+1}}\
            &=[x^{n-k}](1-x)^{-k-1}tag{1}\
            &=[x^{n-k}]sum_{j=0}^infty binom{-k-1}{j}(-x)^jtag{2}\
            &=[x^{n-k}]sum_{j=0}^infty binom{k+j}{j}x^jtag{3}\
            &=binom{n}{n-k}tag{4}\
            &,,color{blue}{=binom{n}{k}}tag{5}
            end{align*}



            and the claim follows.




            Comment:




            • In (1) we apply the rule $[x^{p-q}]A(x)=[x^p]x^qA(x)$.


            • In (2) we apply the binomial series expansion.


            • In (3) we use the binomial identity $binom{-p}{q}=binom{p+q-1}{q}(-1)^q$.


            • In (4) we select the coefficient of $x^{n-k}$.


            • In (5) we use the binomial identity $binom{p}{q}=binom{p}{p-q}$.







            share|cite|improve this answer









            $endgroup$


















              2












              $begingroup$

              Here is a variation based upon the coefficient of operator $[x^n]$ which denotes the coefficient of $x^n$ of a series.




              We obtain
              begin{align*}
              color{blue}{b_n=[x^n]f(x)}&=[x^n]frac{x^k}{(1-x)^{k+1}}\
              &=[x^{n-k}](1-x)^{-k-1}tag{1}\
              &=[x^{n-k}]sum_{j=0}^infty binom{-k-1}{j}(-x)^jtag{2}\
              &=[x^{n-k}]sum_{j=0}^infty binom{k+j}{j}x^jtag{3}\
              &=binom{n}{n-k}tag{4}\
              &,,color{blue}{=binom{n}{k}}tag{5}
              end{align*}



              and the claim follows.




              Comment:




              • In (1) we apply the rule $[x^{p-q}]A(x)=[x^p]x^qA(x)$.


              • In (2) we apply the binomial series expansion.


              • In (3) we use the binomial identity $binom{-p}{q}=binom{p+q-1}{q}(-1)^q$.


              • In (4) we select the coefficient of $x^{n-k}$.


              • In (5) we use the binomial identity $binom{p}{q}=binom{p}{p-q}$.







              share|cite|improve this answer









              $endgroup$
















                2












                2








                2





                $begingroup$

                Here is a variation based upon the coefficient of operator $[x^n]$ which denotes the coefficient of $x^n$ of a series.




                We obtain
                begin{align*}
                color{blue}{b_n=[x^n]f(x)}&=[x^n]frac{x^k}{(1-x)^{k+1}}\
                &=[x^{n-k}](1-x)^{-k-1}tag{1}\
                &=[x^{n-k}]sum_{j=0}^infty binom{-k-1}{j}(-x)^jtag{2}\
                &=[x^{n-k}]sum_{j=0}^infty binom{k+j}{j}x^jtag{3}\
                &=binom{n}{n-k}tag{4}\
                &,,color{blue}{=binom{n}{k}}tag{5}
                end{align*}



                and the claim follows.




                Comment:




                • In (1) we apply the rule $[x^{p-q}]A(x)=[x^p]x^qA(x)$.


                • In (2) we apply the binomial series expansion.


                • In (3) we use the binomial identity $binom{-p}{q}=binom{p+q-1}{q}(-1)^q$.


                • In (4) we select the coefficient of $x^{n-k}$.


                • In (5) we use the binomial identity $binom{p}{q}=binom{p}{p-q}$.







                share|cite|improve this answer









                $endgroup$



                Here is a variation based upon the coefficient of operator $[x^n]$ which denotes the coefficient of $x^n$ of a series.




                We obtain
                begin{align*}
                color{blue}{b_n=[x^n]f(x)}&=[x^n]frac{x^k}{(1-x)^{k+1}}\
                &=[x^{n-k}](1-x)^{-k-1}tag{1}\
                &=[x^{n-k}]sum_{j=0}^infty binom{-k-1}{j}(-x)^jtag{2}\
                &=[x^{n-k}]sum_{j=0}^infty binom{k+j}{j}x^jtag{3}\
                &=binom{n}{n-k}tag{4}\
                &,,color{blue}{=binom{n}{k}}tag{5}
                end{align*}



                and the claim follows.




                Comment:




                • In (1) we apply the rule $[x^{p-q}]A(x)=[x^p]x^qA(x)$.


                • In (2) we apply the binomial series expansion.


                • In (3) we use the binomial identity $binom{-p}{q}=binom{p+q-1}{q}(-1)^q$.


                • In (4) we select the coefficient of $x^{n-k}$.


                • In (5) we use the binomial identity $binom{p}{q}=binom{p}{p-q}$.








                share|cite|improve this answer












                share|cite|improve this answer



                share|cite|improve this answer










                answered Jan 26 at 18:52









                Markus ScheuerMarkus Scheuer

                62.6k459149




                62.6k459149























                    1












                    $begingroup$

                    HINT:



                    Inductive hypothesis
                    begin{eqnarray*}
                    f_k(x)= sum_{n=k}^{infty} binom{n}{k} x^n = frac{x^k}{(1-x)^{k+1}}
                    end{eqnarray*}

                    and use
                    begin{eqnarray*}
                    binom{n}{k} + binom{n}{k+1}= binom{n+1}{k+1}.
                    end{eqnarray*}



                    More detail available on request.



                    Further Hint :



                    begin{eqnarray*}
                    f_{k+1}(x) &=& sum_{n=k+1}^{infty} binom{n}{k+1} x^n = sum_{n=k}^{infty} binom{n+1}{k+1} x^{n+1} \
                    &=& sum_{n=k}^{infty} binom{n}{k} x^{n+1} + sum_{n=k}^{infty} binom{n}{k+1} x^{n+1} \ cdots
                    end{eqnarray*}






                    share|cite|improve this answer











                    $endgroup$













                    • $begingroup$
                      From this I got $f_{k+1}(x)=frac{x^k}{(1-x)^{k+2}}$ so $f_{k+1}(x)=frac{1}{(1-x)}f_k(x)$ I never used induction on generating functions so I'm not sure what I'm supposed to do or if I did this correctly
                      $endgroup$
                      – Spasoje Durovic
                      Jan 25 at 16:39


















                    1












                    $begingroup$

                    HINT:



                    Inductive hypothesis
                    begin{eqnarray*}
                    f_k(x)= sum_{n=k}^{infty} binom{n}{k} x^n = frac{x^k}{(1-x)^{k+1}}
                    end{eqnarray*}

                    and use
                    begin{eqnarray*}
                    binom{n}{k} + binom{n}{k+1}= binom{n+1}{k+1}.
                    end{eqnarray*}



                    More detail available on request.



                    Further Hint :



                    begin{eqnarray*}
                    f_{k+1}(x) &=& sum_{n=k+1}^{infty} binom{n}{k+1} x^n = sum_{n=k}^{infty} binom{n+1}{k+1} x^{n+1} \
                    &=& sum_{n=k}^{infty} binom{n}{k} x^{n+1} + sum_{n=k}^{infty} binom{n}{k+1} x^{n+1} \ cdots
                    end{eqnarray*}






                    share|cite|improve this answer











                    $endgroup$













                    • $begingroup$
                      From this I got $f_{k+1}(x)=frac{x^k}{(1-x)^{k+2}}$ so $f_{k+1}(x)=frac{1}{(1-x)}f_k(x)$ I never used induction on generating functions so I'm not sure what I'm supposed to do or if I did this correctly
                      $endgroup$
                      – Spasoje Durovic
                      Jan 25 at 16:39
















                    1












                    1








                    1





                    $begingroup$

                    HINT:



                    Inductive hypothesis
                    begin{eqnarray*}
                    f_k(x)= sum_{n=k}^{infty} binom{n}{k} x^n = frac{x^k}{(1-x)^{k+1}}
                    end{eqnarray*}

                    and use
                    begin{eqnarray*}
                    binom{n}{k} + binom{n}{k+1}= binom{n+1}{k+1}.
                    end{eqnarray*}



                    More detail available on request.



                    Further Hint :



                    begin{eqnarray*}
                    f_{k+1}(x) &=& sum_{n=k+1}^{infty} binom{n}{k+1} x^n = sum_{n=k}^{infty} binom{n+1}{k+1} x^{n+1} \
                    &=& sum_{n=k}^{infty} binom{n}{k} x^{n+1} + sum_{n=k}^{infty} binom{n}{k+1} x^{n+1} \ cdots
                    end{eqnarray*}






                    share|cite|improve this answer











                    $endgroup$



                    HINT:



                    Inductive hypothesis
                    begin{eqnarray*}
                    f_k(x)= sum_{n=k}^{infty} binom{n}{k} x^n = frac{x^k}{(1-x)^{k+1}}
                    end{eqnarray*}

                    and use
                    begin{eqnarray*}
                    binom{n}{k} + binom{n}{k+1}= binom{n+1}{k+1}.
                    end{eqnarray*}



                    More detail available on request.



                    Further Hint :



                    begin{eqnarray*}
                    f_{k+1}(x) &=& sum_{n=k+1}^{infty} binom{n}{k+1} x^n = sum_{n=k}^{infty} binom{n+1}{k+1} x^{n+1} \
                    &=& sum_{n=k}^{infty} binom{n}{k} x^{n+1} + sum_{n=k}^{infty} binom{n}{k+1} x^{n+1} \ cdots
                    end{eqnarray*}







                    share|cite|improve this answer














                    share|cite|improve this answer



                    share|cite|improve this answer








                    edited Jan 25 at 16:52

























                    answered Jan 25 at 16:08









                    Donald SplutterwitDonald Splutterwit

                    22.9k21446




                    22.9k21446












                    • $begingroup$
                      From this I got $f_{k+1}(x)=frac{x^k}{(1-x)^{k+2}}$ so $f_{k+1}(x)=frac{1}{(1-x)}f_k(x)$ I never used induction on generating functions so I'm not sure what I'm supposed to do or if I did this correctly
                      $endgroup$
                      – Spasoje Durovic
                      Jan 25 at 16:39




















                    • $begingroup$
                      From this I got $f_{k+1}(x)=frac{x^k}{(1-x)^{k+2}}$ so $f_{k+1}(x)=frac{1}{(1-x)}f_k(x)$ I never used induction on generating functions so I'm not sure what I'm supposed to do or if I did this correctly
                      $endgroup$
                      – Spasoje Durovic
                      Jan 25 at 16:39


















                    $begingroup$
                    From this I got $f_{k+1}(x)=frac{x^k}{(1-x)^{k+2}}$ so $f_{k+1}(x)=frac{1}{(1-x)}f_k(x)$ I never used induction on generating functions so I'm not sure what I'm supposed to do or if I did this correctly
                    $endgroup$
                    – Spasoje Durovic
                    Jan 25 at 16:39






                    $begingroup$
                    From this I got $f_{k+1}(x)=frac{x^k}{(1-x)^{k+2}}$ so $f_{k+1}(x)=frac{1}{(1-x)}f_k(x)$ I never used induction on generating functions so I'm not sure what I'm supposed to do or if I did this correctly
                    $endgroup$
                    – Spasoje Durovic
                    Jan 25 at 16:39




















                    draft saved

                    draft discarded




















































                    Thanks for contributing an answer to Mathematics Stack Exchange!


                    • Please be sure to answer the question. Provide details and share your research!

                    But avoid



                    • Asking for help, clarification, or responding to other answers.

                    • Making statements based on opinion; back them up with references or personal experience.


                    Use MathJax to format equations. MathJax reference.


                    To learn more, see our tips on writing great answers.




                    draft saved


                    draft discarded














                    StackExchange.ready(
                    function () {
                    StackExchange.openid.initPostLogin('.new-post-login', 'https%3a%2f%2fmath.stackexchange.com%2fquestions%2f3087246%2ffinding-the-closed-form-of-a-generating-function%23new-answer', 'question_page');
                    }
                    );

                    Post as a guest















                    Required, but never shown





















































                    Required, but never shown














                    Required, but never shown












                    Required, but never shown







                    Required, but never shown

































                    Required, but never shown














                    Required, but never shown












                    Required, but never shown







                    Required, but never shown







                    Popular posts from this blog

                    Mario Kart Wii

                    What does “Dominus providebit” mean?

                    Antonio Litta Visconti Arese